Difference between revisions of "2012 USAMO Problems/Problem 1"

(Solution)
Line 12: Line 12:
  
 
Now since we know that the Fibonacci set is the smallest possible set which does not satisfy the conditions of the problem, then any set <math>\{c\}</math> whose greatest term is less than <math>F_{n-1}</math> must satisfy the conditions. And since <math>\{c\}</math> is bounded between <math>1</math> and <math>n^2</math>, then the conditions of the problem are met if and only if <math>F_{n-1} > n^2</math>. The first <math>n</math> for which this restriction is satisfied is <math>n=13</math> and the exponential behavior of the Fibonacci numbers ensure that every <math>n</math> greater than <math>13</math> will also satisfy this restriction. So the final solution set is <math>\boxed{\{n \ge 13\}}</math>.
 
Now since we know that the Fibonacci set is the smallest possible set which does not satisfy the conditions of the problem, then any set <math>\{c\}</math> whose greatest term is less than <math>F_{n-1}</math> must satisfy the conditions. And since <math>\{c\}</math> is bounded between <math>1</math> and <math>n^2</math>, then the conditions of the problem are met if and only if <math>F_{n-1} > n^2</math>. The first <math>n</math> for which this restriction is satisfied is <math>n=13</math> and the exponential behavior of the Fibonacci numbers ensure that every <math>n</math> greater than <math>13</math> will also satisfy this restriction. So the final solution set is <math>\boxed{\{n \ge 13\}}</math>.
 +
==This is not graffiti==
 +
Uh, <math>F_12</math>=144 and 13^2=169. Last I checked, 169>144
  
 
==See also==
 
==See also==

Revision as of 11:23, 28 April 2012

Problem

Find all integers $n \ge 3$ such that among any $n$ positive real numbers $a_1$, $a_2$, $\dots$, $a_n$ with \[\max(a_1, a_2, \dots, a_n) \le n \cdot \min(a_1, a_2, \dots, a_n),\] there exist three that are the side lengths of an acute triangle.

Solution

Without loss of generality, assume that the set $\{a\}$ is ordered from least to greatest so that the bounding condition becomes $a_n \le n \cdot a_1.$ Now set $b_i \equiv \frac{a_i}{a_1},$ and since a triangle with sidelengths from $\{a\}$ will be similar to the corresponding triangle from $\{b\},$ we simply have to show the existence of acute triangles in $\{b\}.$ Note that $b_1 = 1$ and for all $i$, $b_i \le n.$

Now three arbitrary sidelengths $x$, $y$, and $z$, with $x \le y \le z,$ will form a valid triangle if and only if $x+y>z.$ Furthermore, this triangle will be acute if and only if $x^2 + y^2 > z^2.$ However, the first inequality can actually be inferred from the second, since $x+y>z \longrightarrow x^2 + y^2 +2xy > z^2$ and $2xy$ is trivially greater than $0.$ So we just need to find all $n$ such that there is necessarily a triplet of $b$'s for which $b_i^2 + b_j^2 > b_k^2$ (where $b_i < b_j < b_k$).

We now make another substitution: $c_i \equiv b_i ^2.$ So $c_1 = 1$ and for all $i$, $c_i \le n^2.$ Now we examine the smallest possible sets $\{c\}$ for small $n$ for which the conditions of the problem are not met. Note that by smallest, we mean the set whose greatest element is as small as possible. If $n=3$, then the smallest possible set, call it $\{s_3\},$ is trivially $\{1,1,2\}$, since $c_1$ and $c_2$ are obviously minimized and $c_3$ follows as minimal. Using this as the base case, we see inductively that in general $\{s_n\}$ is the set of the first $n$ Fibonacci numbers. To show this note that if $\{s_n\} = \{F_0, F_1, ... F_n\}$, then $\{s_{n+1}\} = \{F_0, F_1, ... F_n, c_{n+1}\}.$ The smallest possible value for $c_{n+1}$ is the sum of the two greatest values of $\{s_n\}$ which are $F_{n-1}$ and $F_n$. But these sum to $F_{n+1}$ so $\{s_{n+1}\} = \{F_0, F_1, ... F_{n+1}\}$ and our induction is complete.

Now since we know that the Fibonacci set is the smallest possible set which does not satisfy the conditions of the problem, then any set $\{c\}$ whose greatest term is less than $F_{n-1}$ must satisfy the conditions. And since $\{c\}$ is bounded between $1$ and $n^2$, then the conditions of the problem are met if and only if $F_{n-1} > n^2$. The first $n$ for which this restriction is satisfied is $n=13$ and the exponential behavior of the Fibonacci numbers ensure that every $n$ greater than $13$ will also satisfy this restriction. So the final solution set is $\boxed{\{n \ge 13\}}$.

This is not graffiti

Uh, $F_12$=144 and 13^2=169. Last I checked, 169>144

See also

2012 USAMO (ProblemsResources)
First Problem Followed by
Problem 2
1 2 3 4 5 6
All USAMO Problems and Solutions